[Math] Is a norm closed set(in the topology induced by the norm) weakly closed

functional-analysis

My attempt:Yes because since $T_{\text{Norm}} \supset T_{\text{Weak}} \implies T_{\text{Norm}}^{C} \subset T_{\text{Weak}}^C$ Right? Or have I got something wrong here?

This first set inclusion holds because weak topology is defined as the topology which makes all the functionals on the dual space continuous which are already continuous by definition in the norm topology

Best Answer

I think the answer is no. At the very least, it is not true that the set of closed sets is the complement of the set of open sets. Instead, the closed sets are the complements of the open sets, which is different. Indeed, adding open sets also adds closed sets!

The example I have in mind is $X=L^2([0,\pi])$, where $\sin(nx)$ is a sequence which is contained on a sphere and converges weakly to zero, which is not on the sphere. Thus I think that the sphere cannot be weakly closed, but it is certainly strongly closed (the norm is strongly continuous and singletons are closed in $\mathbb{R}$).